• This is an assessment test.
  • To draw maximum benefit, study the concepts for the topic concerned.
  • Kindly take the tests in this series with a pre-defined schedule.

Algebra: Functions Test-3

Congratulations - you have completed Algebra: Functions Test-3.You scored %%SCORE%% out of %%TOTAL%%.You correct answer percentage: %%PERCENTAGE%% .Your performance has been rated as %%RATING%%
Your answers are highlighted below.
Question 1
Largest value of
$ \displaystyle \min \left( 2+{{x}^{2}},6-3x \right)when\,\,x>0$
A
1
B
2
C
3
D
4
Question 1 Explanation: 
$ \displaystyle \begin{array}{l}Equating\,\,\,\,2+{{x}^{2}}=6-3x\\\Rightarrow \,\,\,{{x}^{2}}+3x-4=0\Rightarrow {{x}^{2}}+4x-x-4=0\\or\,\,\,\left( x+4 \right)\,\left( x-1 \right)=0\\\Rightarrow \,x=-4\,\,\,or\,\,\,1\\But\,\,\,x>0\,\,\,so\,\,\,x=1,\,\,\,so\,\,\,LHS=RHS=2+1=3\end{array}$
Capture TS
Therefore we can conclude that the largest value of the function when x>0 occurs at x=1.
The value of y for x=1 is 3. Therefore option (c).
Question 2
A, S, M and D are functions of x and y, and they are defined as follows:
$ \displaystyle \begin{array}{l}A\,\left( x,y \right)=x+y\\S\,\left( x,y \right)=x-y\\M\,\left( x,y \right)=xy\\D\,\left( x,y \right)=x/y\,\,\,\,\,\,\,\,\,\,\,\,\,\,\,\,\,\,\,\,\,\,\,\,\,\,\,\,\,\,\,\,\,\,\,\,\,where\,y\ne 0.\end{array}$
What is the value of
$ \displaystyle M\,\left( M\,(A\,(M\,\left( x,y \right),S\left( y,x \right),x),A\,\left( y,x \right) \right)\,\,\,\,for\,\,\,x=2,\,y=3$
A
50
B
140
C
25
D
70
Question 2 Explanation: 
$ \displaystyle \begin{array}{l}M\,\left( M\,(A\,(M\,\left( x,y \right),S\left( y,x \right),x),A\,\left( y,x \right) \right)\\M\left( M\left( A\left( 6,1 \right),2 \right),A\left( 3,2 \right) \right)\\M\left( M\left( 7,2 \right),A\left( 3,2 \right) \right)\\M\left( 14,5 \right)=70\end{array}$
Question 3
A, S, M and D are functions of x and y, and they are defined as follows: $ \displaystyle \begin{array}{l}A\,\left( x,y \right)=x+y\\S\,\left( x,y \right)=x-y\\M\,\left( x,y \right)=xy\\D\,\left( x,y \right)=x/y\,\,\,\,\,\,\,\,\,\,\,\,\,\,\,\,\,\,\,\,\,\,\,\,\,\,\,\,\,\,\,\,\,\,\,\,\,where\,y\ne 0.\end{array}$
What is the value of
$ \displaystyle S\,\left[ M\,\left( D\,(A\,\left( a,b \right),2 \right),\,D\,\left( A\,\left( a,b \right),2) \right),\,(M(D\,(S\left( a,b \right),2),\,D(S\,\left( a,b \right),2))\, \right]$  
A
a2+b2
B
ab
C
a2-b2
D
a/b
Question 3 Explanation: 
$ \displaystyle \begin{array}{l}S\,\left[ M\,\left( D\,(A\,\left( a,b \right),2 \right),\,D\,\left( A\,\left( a,b \right),2) \right),\,(M(D\,(S\left( a,b \right),2),\,D(S\,\left( a,b \right),2))\, \right]\\=S\,\left[ M\,\left( a+b)/2 \right),\,\left( a+b)/2) \right),\,(M((a-b)/2,\,(a-b)/2))\, \right]\\=S\,\left[ {{(a+b)}^{2}}/4,{{(a-b)}^{2}}/4\, \right]\\={{(a+b)}^{2}}/4+{{(a-b)}^{2}}/4\,\\=4ab/4\\=ab\end{array}$
Question 4
The following functions have been defined:
$ \displaystyle \begin{array}{l}la\,\,\left( x,y,z \right)\,=\min \,\left( x+y,y+z \right)\\le\,\,\left( x,y,z \right)\,=\max \,\left( x-y,y-z \right)\\ma\,\,\left( x,y,z \right)\,=\left( 1/2 \right)\,\left[ le\,\left( x,y,z \right)+la\,\left( x,y,z \right) \right]\end{array}$
Given that $ \displaystyle x>y>z>0$
Which of the following is necessarily true?
A
$ \displaystyle la\,\left( x,y,z \right)<\,le\,\left( x,y,z \right)$
B
$ \displaystyle ma\,\left( x,y,z \right)<\,la\,\left( x,y,z \right)$
C
$ \displaystyle ma\,\left( x,y,z \right)<\,le\,\left( x,y,z \right)$
D
None of these
Question 4 Explanation: 
Taking values as x= 40, y =30, z=2.
la(x,y,z)= min (70,32) =32
le(x,y,z)=max (10,28)=28
ma(x,y,z)=1/2[32+28]=1/2[60]=30
Options (a) and (c) can be easily eliminated by taking random value of x and y (close and very much larger than z).
Taking values as x= 100, y =2, z=1.
la(x,y,z)= min (102,3) =3
le(x,y,z)=max (98,1)=98
ma(x,y,z)=1/2[3+98]=1/2[101]=55.5
Here when we take a >> b>c we find that
ma(x,y,z) > la(x,y,z)= min (102,3) =3
Question 5
The following functions have been defined: $ \displaystyle \begin{array}{l}la\,\,\left( x,y,z \right)\,=\min \,\left( x+y,y+z \right)\\le\,\,\left( x,y,z \right)\,=\max \,\left( x-y,y-z \right)\\ma\,\,\left( x,y,z \right)\,=\left( 1/2 \right)\,\left[ le\,\left( x,y,z \right)+la\,\left( x,y,z \right) \right]\end{array}$
What is the value of
$ \displaystyle ma\,\left( 10,\,4,\,le\,(\,la\,\left( 10,5,3 \right),5,3) \right)$
A
7.0
B
6.5
C
8.0
D
7.5
Question 5 Explanation: 
=$\displaystyle ma\,\left( 10,\,4,\,le\,(\,\min \,\left( 15,8 \right),5,3) \right)$
= ma (10,4, max (8-5,5-3))
= ma (10,4, max (3,2))
= ma (10,4,3)
=1/2 [{la (10,4,3)}+(le (10,4,3)}]
=1/2 [{min (14,7)}+(max(6,1)}]
=1/2 [7+6]
=6.5
Once you are finished, click the button below. Any items you have not completed will be marked incorrect. Get Results
There are 5 questions to complete.
List
Return
Shaded items are complete.
12345
End
Return

Want to explore more Arithmetic Tests?

Explore Our Arithmetic Tests

Website Popup

Join our Free TELEGRAM GROUP for exclusive content and updates

Rsz 1rsz Close Img

Join Our Newsletter

Get the latest updates from our side, including offers and free live updates, on email.

Rsz Undraw Envelope N8lc Smal
Rsz 1rsz Close Img